Is Consistency in Linear Systems Determined by Augmented Matrices?

Click For Summary
The discussion focuses on the consistency of linear systems as determined by augmented matrices. The key point is that a linear system Ax=b is consistent if the rank of the coefficient matrix U equals the rank of the augmented matrix [U|c]. The confusion arises around understanding the "if" part of the statement, which relates to the ranks of the matrices. It is clarified that the condition holds true when the ranks are equal and do not exceed the number of equations. Ultimately, the participants confirm their understanding of the relationship between the ranks and the consistency of the system.
SylowTheorems
Messages
4
Reaction score
0

Homework Statement


Quote from my textbook:

"The linear system Ax=b is consistent if and only if the number of nonzero rows of the augmented matrix [U| c], equals the number of nonzero rows in U."

[U,c] is the rref of [A,b]


Homework Equations





The Attempt at a Solution



I understand "only if" but not "if". "Only if" is true since the elementary row operations do not affect the solutions to the system and it is clear that a linear combination of zeros cannot equal a nonzero number. Can someone help me with "if"?
 
Physics news on Phys.org
number of nonzero rows in U is the rank of U,r(U)
the no. of nonzero rows of [U|c] is the rank of that matrix,r(U|c)

For a system of eq'ns to be constisent

r(U)=r(U|c) \leq n

n= no . of rows
 
Thanks for responding to my question!

Unfortunately, I am still confused. It seems like that just restated my question using the word "rank".


rock.freak667 said:
For a system of eq'ns to be constisent

r(U)=r(U|c) \leq n

I think the quote I gave came from a proof of this statement actually...
 
well saying "if and only if", like restricting the statement so that the statement will be true when that condition is satisfied.
 
I figured it out. Thanks for your help.
 
Question: A clock's minute hand has length 4 and its hour hand has length 3. What is the distance between the tips at the moment when it is increasing most rapidly?(Putnam Exam Question) Answer: Making assumption that both the hands moves at constant angular velocities, the answer is ## \sqrt{7} .## But don't you think this assumption is somewhat doubtful and wrong?

Similar threads

  • · Replies 2 ·
Replies
2
Views
2K
  • · Replies 2 ·
Replies
2
Views
2K
Replies
4
Views
2K
  • · Replies 3 ·
Replies
3
Views
1K
  • · Replies 5 ·
Replies
5
Views
3K
  • · Replies 13 ·
Replies
13
Views
3K
  • · Replies 25 ·
Replies
25
Views
3K
  • · Replies 2 ·
Replies
2
Views
3K
  • · Replies 9 ·
Replies
9
Views
2K
  • · Replies 13 ·
Replies
13
Views
3K